Please confirm topic selection

Are you sure you want to trigger topic in your Anconeus AI algorithm?

Please confirm action

You are done for today with this topic.

Would you like to start learning session with this topic items scheduled for future?

Review Question - QID 215004

In scope icon M 6 A
QID 215004 (Type "215004" in App Search)
A 75-year-old man presents to the emergency department with worsening dyspnea on exertion and fatigue. He states that his symptoms have been worsening for the past month. He denies any baseline or exertional chest pain but states that his exercise tolerance has decreased. He was previously able to walk 4 blocks but now can only walk 1 block. He also endorses weakness and muscle cramps which he has not had before. He saw his primary care doctor 10 days ago who increased his dose of furosemide. The patient states that he has been urinating profusely and has lost 7 pounds over 1 week as a result. He has a past medical history of heart failure, hypertension, diabetes, chronic kidney disease, and hypothyroidism. He takes aspirin, metoprolol, furosemide, hydrochlorothiazide, levothyroxine, atorvastatin, and fish oil. He states he has started drinking 2 cups instead of 1 cup of coffee per day for his decreased energy and tried an extra dose of his thyroid medication for the past 2 days with no improvement in his symptoms. His temperature is 99.5°F (37.5°C), blood pressure is 110/62 mmHg, pulse is 104/min, and respirations are 15/min. Physical exam reveals an overweight man in no acute distress. When he walks, his oxygen saturation drops to 95% from 100%. An ECG is performed as seen in Figure A. Which of the following is the most likely etiology of this patient's ECG finding?
  • A
  • A